Residue Theory for Solving Complex Contour Integrals | Step-by-Step Guide

  • Thread starter Thread starter everjordan
  • Start date Start date
  • Tags Tags
    Theory
Click For Summary
The discussion focuses on solving a complex contour integral using residue theory, specifically for the integral of (z-i)^2 divided by sin^2(z) over a rectangular contour. Participants suggest expanding 1/sin(z) into its Laurent series and multiplying it with (z-i)^2 to extract residues from the z^-1 term. There is uncertainty about the correctness of this approach, but it is noted that similar expansions should be done for other poles within the contour. The conversation emphasizes the importance of identifying the order of the pole at zero and calculating the corresponding residue. The method outlined is a valid strategy for evaluating the integral using residue theory.
everjordan
Messages
7
Reaction score
0

Homework Statement



\oint (z-i)^2 \div sin^2(z)
gamma : 4+4i, 4-4i, -4+4i, -4-4i rectangluar

Homework Equations



residue theory

The Attempt at a Solution



I don't know how to solve this problem.
I do my best using taylor series... however it doesn't solve the problem..
Please help me...
 
Last edited:
Physics news on Phys.org
Expand 1/sin z in its Laurent series and after multiplying the various terms together, pull the residues off the 1/z term.

Hint: what order pole does 1/sin z have? what's the residue of 1/sin z at 0?
 
thank you for reply :)

first, i expanded csc^2(z) in Laurent series

and (z-i)^2 = z^2 -2iz -1

after that, multiply Laurent series and z^2-2iz-1

and pull the residues off the z^-1 term.

but I wasn't sure this was a good idea.

Is this right?
 
Now you just need to do the same thing for the other poles inside your contour, except you have to expand the series around those points instead. (so the series will be in (z-pi) and (z+pi) instead of just z).
 
Question: A clock's minute hand has length 4 and its hour hand has length 3. What is the distance between the tips at the moment when it is increasing most rapidly?(Putnam Exam Question) Answer: Making assumption that both the hands moves at constant angular velocities, the answer is ## \sqrt{7} .## But don't you think this assumption is somewhat doubtful and wrong?

Similar threads

Replies
1
Views
1K
  • · Replies 8 ·
Replies
8
Views
1K
  • · Replies 3 ·
Replies
3
Views
1K
  • · Replies 3 ·
Replies
3
Views
2K
Replies
5
Views
2K
  • · Replies 1 ·
Replies
1
Views
1K
Replies
4
Views
2K
  • · Replies 2 ·
Replies
2
Views
2K
  • · Replies 6 ·
Replies
6
Views
2K
  • · Replies 8 ·
Replies
8
Views
2K